Noetherian Modules and Finitely Generated Modules

  • MHB
  • Thread starter Math Amateur
  • Start date
  • Tags
    Modules
In summary, the conversation discusses proving Theorem 2.2 on Noetherian modules in "Introduction to Ring Theory" by P. M. Cohn. The key to the proof is showing that every module of M being finitely generated implies that M is Noetherian. The conversation includes an explanation and interpretation of a specific part of the proof regarding submodules and a union of chains. The conversation concludes with a compliment to the clear and explicit argument provided.
  • #1
Math Amateur
Gold Member
MHB
3,990
48
I am reading "Introduction to Ring Theory" by P. M. Cohn (Springer Undergraduate Mathematics Series)

In Chapter 2: Linear Algebras and Artinian Rings we find Theorem 2.2 on Noetherian modules. I need help with showing that "every module of M is finitely generated" implies that "M is Noetherian"

Theorem 2 reads as follows:View attachment 3166
https://www.physicsforums.com/attachments/3167In the above text [at the very end of the text - in the argument for \(\displaystyle (d) \Longrightarrow (a)\)] we read:

" … … If \(\displaystyle a_j \in N_{i_j}\) and \(\displaystyle k = \text{max} \{ i_1, \ … \ … \ i_r \}\), then equality holds in our chain from N_k onwards … … "

I do not follow the argument in the above text … indeed, I am having some trouble interpreting the exact meaning of \(\displaystyle a_j \in N_{i_j}\) … …

Can someone please help me to understand the above argument and notation … my apologies to readers for not being able to make my question/confusion clearer …

Hope someone can help.

Peter
 
Physics news on Phys.org
  • #2
Peter said:
I am reading "Introduction to Ring Theory" by P. M. Cohn (Springer Undergraduate Mathematics Series)

In Chapter 2: Linear Algebras and Artinian Rings we find Theorem 2.2 on Noetherian modules. I need help with showing that "every module of M is finitely generated" implies that "M is Noetherian"

Theorem 2 reads as follows:View attachment 3166
https://www.physicsforums.com/attachments/3167In the above text [at the very end of the text - in the argument for \(\displaystyle (d) \Longrightarrow (a)\)] we read:

" … … If \(\displaystyle a_j \in N_{i_j}\) and \(\displaystyle k = \text{max} \{ i_1, \ … \ … \ i_r \}\), then equality holds in our chain from N_k onwards … … "

I do not follow the argument in the above text … indeed, I am having some trouble interpreting the exact meaning of \(\displaystyle a_j \in N_{i_j}\) … …

Can someone please help me to understand the above argument and notation … my apologies to readers for not being able to make my question/confusion clearer …

Hope someone can help.

Peter

Hi Peter,

First, let's start with the ascending chain $N_1 \subseteq N_2 \subseteq \cdots$. We want to show that this chain stabilizes, i.e., there exists some $k$ such that $N_k = N_{k+1} = \cdots$. To do this, we consider the least upper bound of the chain, that is, $N := \cup N_i$. Recall that the union of a chain of an ascending chain of submodules is a submodule; in particular, $N$ is a submodule of $M$. Since it's assumed that every submodule of $M$ is finitely generated, we know that $N$ is finitely generated. That's why Cohn can choose a finite generating set $\{a_1,\ldots, a_r\}$ for $N$. But since $N$ is the union of the $N_i$, we know that each $a_j$ belongs to at least one of the $N_i$. In other words, for each $j\in \{1,2,\ldots, r\}$, there exists an index $i_j$ such that $a_j\in N_{i_j}$. By choosing $k$ to be the largest of the indices $i_1,\ldots, i_r$, you ensure that $N_k = N$, and hence $N_k = N_{k+1} = \cdots$. Make sense?

Just in case you didn't see how $N_k = N$, I'll put in more detail here. Since $N_{i_j} \subseteq N_k$ for all $j$ (by the ascending chain and the fact that $i_j \le k$ for all $j$) and $a_j\in N_{i_j}$ for all $j$, we have $a_j \in N_k$ for all $j$. Hence, the submodule generated by $a_1,\ldots a_r$ is a submodule of $N_k$. But since $N$ is generated by $a_1,\ldots, a_r$, $N \subseteq N_k$. On the other hand, $N_k \subseteq N$ by construction of $N$. Hence, $N_k = N$.
 
  • #3
Euge said:
Hi Peter,

First, let's start with the ascending chain $N_1 \subseteq N_2 \subseteq \cdots$. We want to show that this chain stabilizes, i.e., there exists some $k$ such that $N_k = N_{k+1} = \cdots$. To do this, we consider the least upper bound of the chain, that is, $N := \cup N_i$. Recall that the union of a chain of an ascending chain of submodules is a submodule; in particular, $N$ is a submodule of $M$. Since it's assumed that every submodule of $M$ is finitely generated, we know that $N$ is finitely generated. That's why Cohn can choose a finite generating set $\{a_1,\ldots, a_r\}$ for $N$. But since $N$ is the union of the $N_i$, we know that each $a_j$ belongs to at least one of the $N_i$. In other words, for each $j\in \{1,2,\ldots, r\}$, there exists an index $i_j$ such that $a_j\in N_{i_j}$. By choosing $k$ to be the largest of the indices $i_1,\ldots, i_r$, you ensure that $N_k = N$, and hence $N_k = N_{k+1} = \cdots$. Make sense?

Just in case you didn't see how $N_k = N$, I'll put in more detail here. Since $N_{i_j} \subseteq N_k$ for all $j$ (by the ascending chain and the fact that $i_j \le k$ for all $j$) and $a_j\in N_{i_j}$ for all $j$, we have $a_j \in N_k$ for all $j$. Hence, the submodule generated by $a_1,\ldots a_r$ is a submodule of $N_k$. But since $N$ is generated by $a_1,\ldots, a_r$, $N \subseteq N_k$. On the other hand, $N_k \subseteq N$ by construction of $N$. Hence, $N_k = N$.
Thanks for an extremely clear and explicit argument, Euge … thanks to your post I now follow the proof ...

Indeed, if you are not already teaching algebra at a University … then you should be … ...

Peter
 

1. What is a Noetherian module?

A Noetherian module is a module that satisfies the ascending chain condition on submodules. This means that for every increasing chain of submodules, there exists a finite number of submodules before the chain stabilizes. In other words, there are no infinite increasing chains of submodules in a Noetherian module.

2. How are Noetherian modules related to Noetherian rings?

A Noetherian module is a module over a Noetherian ring. This means that the ring itself satisfies the ascending chain condition on ideals, and thus every submodule of a Noetherian module is also a finitely generated submodule. This relationship is important because Noetherian rings and modules have many useful properties in algebraic geometry and commutative algebra.

3. What is the difference between a finitely generated module and a Noetherian module?

A finitely generated module is a module that can be generated by a finite set of elements. This means that every element in the module can be written as a linear combination of these generators. A Noetherian module, on the other hand, satisfies the ascending chain condition on submodules. While every Noetherian module is finitely generated, the converse is not always true.

4. How do you determine if a module is Noetherian?

To determine if a module is Noetherian, you can use the Hilbert basis theorem. This theorem states that a commutative ring is Noetherian if and only if its polynomial ring over a field is Noetherian. Therefore, to determine if a module is Noetherian, you can check if the ring it is over is Noetherian. Alternatively, you can check if the module satisfies the ascending chain condition on submodules directly.

5. Can a non-finitely generated module be Noetherian?

No, a non-finitely generated module cannot be Noetherian. This is because a module that is not finitely generated will always have an infinite increasing chain of submodules, which violates the ascending chain condition. Therefore, a non-finitely generated module cannot satisfy the definition of a Noetherian module.

Similar threads

  • Linear and Abstract Algebra
Replies
3
Views
1K
  • Linear and Abstract Algebra
Replies
2
Views
969
  • Linear and Abstract Algebra
Replies
3
Views
1K
Replies
10
Views
2K
Replies
6
Views
1K
  • Linear and Abstract Algebra
Replies
13
Views
2K
Replies
2
Views
2K
Replies
1
Views
1K
  • Linear and Abstract Algebra
Replies
9
Views
1K
Replies
9
Views
1K
Back
Top